Dirac-Gleichung und Spin: nichtrelativistischer Grenzfall

From testwiki
Jump to navigation Jump to search

{{#set:Urheber=Prof. Dr. T. Brandes|Inhaltstyp=Script|Kapitel=1|Abschnitt=5}} Kategorie:Quantenmechanik __SHOWFACTBOX__



Mit (Vektor) Potential haben wir die Dirac-Gleichung{{#set:Fachbegriff=Dirac-Gleichung|Index=Dirac-Gleichung}} als

itΨ=(α_(p^_eA_)+βm+eϕ)Ψ,=c=1
     (1.37)


Jetzt erfolgt die Zerlegung Ψ=(φχ)eimtRuheenergie-Phasenfaktor=(φχ)eimc2t, mit den 2er Spinoren

φ=(φ1φ2),χ=(χ1χ2).


Damit folgt dann

it(φχ)=(σ_(p^_eA_)χσ_(p^_eA_)φ)+eϕ(φχ)2mc2(0χ)
     (1.38)


Beachte das jetzt überall φ=φ(x_,t)gilt

Jetzt: Näherung/Annahme das kinetische und potentielle Energie viel kleiner als Ruhemasse mc2 ist

mcχ2|itχ|,mcχ2|eϕφ|χ12mc2σ_(p_eA_)φ
     (1.39)

einsetzen in die Gleichung (1.38) liefert

itφ=12m(σ_(p_eA_)2)φ+eϕφ


     (1.40)


Jetzt folgendes „Theorem“ benutzen

(σ_A_)(σ_B_)=A_B_1__+iσ_(A_×B_)
     (1.41)

mit

A_=(A1,A2,A3),B_=(B1,B2,B3),A_,B_ vektorwertiger Operator undσ_=(σ__1,σ__2,σ__3)Vektor der Pauli-Matrizen

Beweis von (1.41) mittels (Anti) Kommutator-Eigenschaften{{#set:Fachbegriff=Kommutator-Eigenschaften|Index=Kommutator-Eigenschaften}} (AUFGABE)

{σ__i,σ__j}:=σ__iσ__j+σ__jσ__i=2δij1__[σ__i,σ__j]:=σ__iσ__jσ__jσ__i=2iεijkσ__k
     (1.42)


Es gilt weiterhin (AUFGABE), beachte p_=i_ und A_=A_(x_,t)

(p_eA_)×(p_eA_)=ei(_×A_)Magnetfeld=eiB_Magnetfeld      (1.43)


Mit (1.43) folgt aus (1.41) die Kopplung von Spin und Magnetfeld

Pauli-Gleichung{{#set:Fachbegriff=Pauli-Gleichung|Index=Pauli-Gleichung}} itφ=[12m(p_eA_)2e2mσ_.B_Pauli-Term+eϕ]φ


     (1.44)


mit dem 2-Komponentigen Spinor φ=(φ1φ2)

siehe auch[edit | edit source]

Der_nichtrelativistische_Grenzfall#Nichtrelativistische_Näherung:

Literatur

LITERATUR: GREINER